PNP Practice Test 2

अब Quizwiz के साथ अपने होमवर्क और परीक्षाओं को एस करें!

A family who recently immigrated present with concerns about their infant's fever and rhinitis. Which of the following questions BEST elicits the family's cultural perspective of the illness? A. "What do you think has caused this problem?" B. "Has your baby been in contact with other sick children?" C. "What do you think is the best way to feed your baby during illness?" D. "Has your baby ever been sick like this before the family immigrated?"

The correct answer is A - "What do you think has caused this problem?" Beliefs, values, and views of health and illness are very different across cultural groups. When working with families from different cultures, the challenge can be finding a mutual understanding for the plan of care. Eliciting a client's ideas about the cause of illness in a culturally respectful manner is the first step in negotiating a mutually agreed upon plan. Asking if the infant has been around other sick children assumes the family adheres to the germ theory, while asking whether the infant was ill before the family immigrated seeks information, but not about culture. When posing the question about feeding while the infant is sick can vary based on culture, it does not address cultural beliefs about the illness itself

A 10-month-old male is irritable and has lower extremity weakness on physical exam. Which of the following infant characteristics would prompt screening for rickets? A. African American ethnicity B. living below 40 degrees latitude C. recent relocation from Texas to Wisconsin D. breastfed supplemented with 18 oz (532 mL) formula per day

The correct answer is A - African American ethnicity Rickets is not a disease of the past nor is it limited to developing countries. Children of darker-skinned ethnicity are at increased risk for rickets as more melanin decreases UV-B penetration, so they require longer exposure to sunlight to achieve adequate vitamin D levels. Dark-skinned infants who are exclusively breastfed and infants born to mothers who were vitamin D deficient through pregnancy are at higher risk for developing rickets. The American Academy of Pediatrics (AAP) recommends that breastfed infants receive 400 IU of vitamin D as a supplement until they are receiving at least 1000 mL/day of formula or whole milk. Although rickets may occur in children living at lower latitudes, it is more common in children in the northern United States, above 40 degrees latitude

An adolescent has a 4-week history of a paroxysmal cough and occasional post-tussive emesis. Physical exam is essentially unremarkable except for signs of an upper respiratory infection. The MOST likely causative organism is: A. Bordetella pertussis B. Haemophilus influenzae C. Mycoplasma pneumoniae D. Streptococcus pneumoniae

The correct answer is A - Bordetella pertussis Pertussis should be suspected in any individual who has a persistent cough especially in the absence of fever, malaise, myalgia, tachypnea, wheezes or rales. Pertussis is a strong possibility if the cough has lasted more than 14 days, with associated paroxysms, whoops or post-tussive emesis. The characteristic whooping-type cough is often absent in adults or adolescents. In adolescents, pertussis is considered when a persistent cough is present for 3 weeks. Pneumonia, due to Streptococcus pneumoniae, also may cause post-tussive emesis, but is usually accompanied by a history of fever and headache along with rales heard on auscultation. Haemophilus influenzae, as a cause of pneumonia, is more common in younger children less than 4 years of age. The cough associated with Mycoplasma pneumoniae is usually resolving by the second week of the illness and may have some accompanying wheezing.

A nurse practitioner is purchasing a "claims-based malpractice insurance policy" for the first time. This will cover A. a claim made during the period the policy is active depending on state laws and age of claimant. B. any event that occurs during the policy period regardless of when the claim is made. C. an extension of the original malpractice policy if new employment occurs. D. a claim made prior to the inception date of the policy.

The correct answer is A - a claim made during the period the policy is active depending on state laws and age of claimant. Nurse practitioners (NP) need to know the type of malpractice insurance which is applicable during their employment. Occurrence-based malpractice insurance covers any event that "occurs" during the policy period regardless of when a claim is made. Claims-based malpractice insurance provides coverage for a claim that is made during the period when the policy is / was active. Claims may be made many years after a medical event, depending on state laws and the age of claimant at the time of the event, but this type of coverage would not cover claims outside of the policy period. Malpractice coverage referred to as "nose" coverage, can be attached to a claims-made policy once the NP is under new employment. Claims-made liability policies usually include a retroactive date, and the policy will not cover claims for what may have occurred before the retroactive date. "Tail" malpractice insurance extends the policy and can be provided by the previous employer or by the NP. The claim must be made while the NP is still employed by the previous practice, and the NP must be notified of the claim while the NP is still working on behalf of that practice

Completion of a urine drug screening for an adolescent at the request of a parent is considered A. a risk to the patient-practitioner relationship. B. unreliable because of the long half-life of most drugs of abuse. C. an acceptable measure to assist with patient referral for therapy. D. a reliable method of determining recent use of a substance such as marijuana

The correct answer is A - a risk to the patient-practitioner relationship. Although practitioners should complete substance use screening at every adolescent well visit, performing drug screens at the request of parents should be avoided because of the risk of interference with a trusting patient-provider relationship. The testing of urine for drugs of abuse is not always reliable since the accuracy of results can be affected by factors such as the specific gravity and creatinine concentration of the urine. A lower creatinine level can dilute amount of drug in urine. Measurement of urine pH is valuable to check for chemical adulteration. Except in the case of marijuana, stimulants and cocaine have relatively short half-lives, so urine testing only reflects use of these substances within the past 1-3 days. Overall, laboratory testing should not be performed without the knowledge and consent of the competent adolescent

During a sports physical, an otherwise healthy 18-year-old female who is a competitive athlete gives a history of mild abdominal pain and nausea, last menstrual period 3 months ago, and weight loss of 30 pounds in the past year. Which of the following is the MOST likely diagnosis? A. anorexia nervosa B. celiac disease C. inflammatory bowel disease D. new onset diabetes

The correct answer is A - anorexia nervosa The DSM 5 criteria for the diagnosis of anorexia nervosa includes restriction of energy intake relative to requirements, intense fear of gaining weight and disturbance in body image. Amenorrhea is an important clinical sign but was actually removed from the DSM criterion when the DSM 5 was published. Patients with anorexia are often bradycardic because the body becomes functionally euthyroid to save energy. Anorexia nervosa can additionally cause structural and functional cardiovascular changes which manifest as hypotension, left ventricular systolic dysfunction, prolonged QT syndrome and QT dispersion, increasing the risk for arrhythmias. Patients who are diagnosed with diabetes complain of abdominal pain and nausea, but most often present weight loss associated with polydipsia, polyuria, and polyphagia. None of the other diagnoses are consistent with amenorrhea. Pregnancy should always be considered in the setting of amenorrhea in a previously menstruating female but is typically not associated with significant weight loss. By 12 weeks the uterus can typically be palpated just above the symphysis pubis.

A newborn fails the initial hearing screen by otoacoustic emissions. Which method of testing provides the MOST accurate reassessment? A. auditory brainstem response B. behavioral observation audiometry C. repeat otoacoustic emissions D. visual reinforced audiometry

The correct answer is A - auditory brainstem response According to the CDC, hearing loss occurs in about 1.4 out of every 1000 infants. Of infants with hearing loss, the cause is unknown in about 25%. Among known causes, about 50% are related to genetic factors. Other causes are illness and trauma, which include severe stress at birth, viral infections, or meningitis. A hearing screening is recommended for newborns before discharge from the birth hospital and no later than 1 month of age, although programs vary by state. The initial screening is typically an otoacoustic emission test (OAE) which uses a specialized instrument to detect inner ear responses to brief clicks or tones. Between 2 and 10% of newborns have screening results that are out of range that require a complete audiology evaluation. Newborn hearing screening can be affected by factors such as crying or movement during the test, as well as the presence of vernix in the ear canal or fluid in the middle ear. When results of the OAE are positive, auditory brainstem response (ABR) is typically the next step in evaluation. The test, which records brain wave activity in response to sounds, is not a direct assessment of hearing but is correlated to hearing thresholds. The test takes 1-2 hours, and is not affected by sedation when required. Detection of hearing problems can be challenging in young infants who are likely to achieve early language milestones such as smiling, cooing and babbling at the normal times despite hearing impairment, making behavioral interpretation less accurate. Potential signs of a hearing loss include the infant responding upon visual contact with the caregiver as opposed to turning towards a caregiver's voice. Signs of normal hearing development in young infants include startling at loud noises, ceasing activity with conversational sound to include quieting to their mother's voice. Children may develop hearing impairments after the neonatal period, and the difficulties may go undetected until preschool or kindergarten screening occurs. Health care providers should be alert to early signs and symptoms to identify hearing impairments as early as possible.

A child weighing 45 kg has a positive rapid strep test and is started on amoxicillin. Two days later the mother calls and reports the child has developed an itchy, raised rash over his trunk, but no other symptoms. Which of the following is the BEST management strategy? A. change to clindamycin 300 mg every 8 hours for 10 days B. change to minocycline 75 mg twice daily for 10 days with meals C. acknowledge penicillin allergy and add an antihistamine to the current regimen D. continue the amoxicillin as the rash is a response to a toxin from the strep bacteria

The correct answer is A - change to clindamycin 300 mg every 8 hours for 10 days The itchy, raised rash described suggests the child may have an allergic sensitivity to penicillin. For penicillin-allergic individuals, acceptable alternatives include a narrow-spectrum oral cephalosporin, oral clindamycin, or oral macrolides or azalides. Minocycline is used to treat susceptible Gram-negative and Gram-positive infections involving the respiratory or urinary tracts in children over 8 years of age. The oral formulation of clindamycin is prepared as 75 mg/5 ml, and the recommended dose for children with streptococcal pharyngitis is 20 mg/kg/day divided every 8 hours for 10 days.

A previously healthy child's leg wound is cultured in the office with results revealing methicillin-resistant Staphylococcus aureus (MRSA). With a known allergy to sulfa, the BEST choice for treatment is: A. clindamycin (Cleocin) B. rifampin (Rifadin) C. trimethoprim-sulfamethoxazole (Bactrim) D. vancomycin (Vancocin )

The correct answer is A - clindamycin (Cleocin) Methicillin-resistant Staphylococcus aureus (MRSA) is increasingly prevalent as the organism involved in community acquired skin and soft tissue infections. Appropriate treatment and prevention strategies need to be incorporated into care. Trimethoprim-sulfamethoxazole is an effective treatment in mild cases of community acquired MRSA (CA-MRSA), but contains sulfa. Clindamycin is an effective treatment for mild to severe infections due to CA-MRSA and an appropriate choice if a child has a known allergy to trimethoprim-sulfamethoxazole. Use of vancomycin is administered intravenously and therefore reserved for treatment of hospitalized individuals with severe CA-MRSA. Rifampin is usually administered with another antimicrobial agent to reduce the development of resistance. Community resistance patterns must also be factored into the decision to choose a particular antimicrobial agent.

The purpose of administering acidophilus (Lactobacillus) while taking antibiotics is to A. decrease pH in the gastrointestinal tract. B. decrease diarrhea by adding additional fiber. C. enhance the absorption of the antibiotic in the colon. D. treat diarrheal illnesses caused by Clostridium difficile

The correct answer is A - decrease pH in the gastrointestinal tract. Lactobacillus acidophilus is often recommended for children on long term antibiotic therapy. It is a probiotic found in yogurt and manufactured as a supplement, which assists in re-establishing the acidic environment in the gastrointestinal tract, thereby reducing the production of bacteria, fungus, or yeasts. It is helpful in the treatment or prevention of antibiotic-induced diarrhea and vaginal or diaper candida infections. While it does not destroy bacteria, such as Clostridium difficile, it creates an unfavorable environment for continued growth of the bacteria.

A 1 week old has a papular rash, with clustered lesions, which was first noticed by the caregiver three days ago. A few areas appear to be pustular. The infant is otherwise healthy. What is the MOST likely diagnosis? A. erythema toxicum B. milia C. neonatal acne D. neonatal pustular melanosis

The correct answer is A - erythema toxicum Erythema toxicum is a common condition in the newborn period which consists of papular, pustular or combined papulopustular lesions from 1 to 3 millimeters in size with surrounding blotchy erythema. It occurs in neonates in the first few days of life up to 2 - 3 weeks of age. Lesions may be clustered, and can recur, but do clear without intervention. Milia is common in newborns and involves white to yellow papules, usually on the upper body and face and can last for several months. Neonatal acne can be present at birth and involves open and closed comedones. Neonatal pustular melanosis is a rare condition, usually occurring in infants with darker skin tones and is a vesicular rash, involving 2 - 5 mm pustules with a hyperpigmented, non-erythematous base. All of these conditions require no treatment.

During a visit for an acute illness, which element of a child's history can only be completed by the health care provider? A. history of present illness B. past medical history C. review of systems D. diagnosis

The correct answer is A - history of present illness With any patient visit, obtaining or completing the history taking can be done in a variety of ways, especially now that many practice settings use an electronic medical record. Ancillary staff, the patient, or parents / caregivers may complete the "review of systems" and "past medical history". However, the provider must complete the "history of the present illness" (HPI) as the HPI supports the evaluation and management reporting. The HPI consists of the historical details of signs and symptoms for the presenting illness as stated by the patient or parent and includes details such as the duration of symptoms, and if anything modifies, relieves, or aggravates the problem. The HPI is used to support the level of evaluation and management needed to comply with coding criteria for billing and later coding audits. The provider makes the diagnosis, but this is not an element of "history". According to the American Academy of Pediatrics (AAP), only the reporting provider can complete the HPI; and the HPI cannot be co-signed

Alternating doses of ibuprofen and acetaminophen for the treatment of fevers A. increases the risk of dosage errors and toxicity. B. allows for lower total dosage of both drugs. C. results in more rapid resolution of the fever than using either drug alone. D. is appropriate only for children over the age of six months

The correct answer is A - increases the risk of dosage errors and toxicity Alternating doses of ibuprofen and acetaminophen for fever is a common practice of parents. Despite the lack of data to support this practice and the potential for harm, some health care providers continue to endorse this practice for their patients. Both acetaminophen and ibuprofen do block prostaglandin synthesis and have the same efficacy. Alternating acetaminophen and ibuprofen may be more effective than either medication alone in reducing fever, but it is unclear if it is clinically significant. There is little information on the effects for a child's discomfort or safety. There is concern for liver and kidney injury, especially in children with volume depletion. The use of ibuprofen is not recommended for infants younger than 6 months. Acetaminophen should be given no more than five times in a 24-hour period, and ibuprofen should be given only every six to eight hours. There is a serious risk of overmedication and possible toxicity with the practice of alternating these medications, and there have been no demonstrable benefits. Therefore, the practice should be discouraged.

Nutritional assessment of an adolescent on a strict vegan diet should include regular growth monitoring, dietary analysis, and laboratory assessment of: A. iron, vitamin B12, and zinc B. calcium, vitamin B6, and zinc C. iron, potassium, and vitamin B6 D. calcium, potassium, and vitamin B12

The correct answer is A - iron, vitamin B12, and zinc Slower growth rates have been reported in infants and children who follow vegan and macrobiotic diets during the first five years of life. However, catch-up growth usually occurs in these children by age 10 years. Strict restriction of dietary fats is not part of a vegan diet, and may provide a clue to serious eating disorders if the adolescent will not eat foods containing fat. Iron deficiency anemia is a common complication of vegan diets in adolescents and will not resolve without dietary change and/or iron supplementation Adolescents who are vegetarian may experience a later onset of menstruation, probably related to a lower percentage of body fat. Strict vegan diets are most lacking in protein and some nutrients. High incidence of deficiencies in vitamin B12, iron, and zinc have been identified and ultimately these children are at risk of developmental delay. Growth monitoring, dietary analysis, counseling, and laboratory assessment of vitamin B12, iron, and zinc should be included in health maintenance visits. Because vegans eat fruits, vegetables, grains, nuts, seeds and legumes, vitamin B6 levels should be within normal limits as sources of vitamin B6 are soybeans, oats, whole wheat, peanuts, and walnuts. Vegans are also typically not nutritionally deficient in potassium as many fruits and vegetables, such as spinach, kale, potatoes, avocados, and potatoes, are potassium-rich

Caregivers report that their child awakens crying and agitated during the last half of the nighttime sleep cycle, followed by appearing fully awake and readily comforted. This child is experiencing: A. nightmares B. sleep starts C. sleep tremors D. somniloquy

The correct answer is A - nightmares Nightmares typically occur during rapid eye movement (REM) sleep in the latter half to one third of the nighttime sleep cycle. In the event of a nightmare, the child is fully awake and responds to parental or caregiver comforting measures. Night terrors or sleep terrors occur in the first portion of sleep and are manifested by abrupt partial awakenings accompanied by a blood curdling scream, characterized by intense autonomic symptoms such as pupillary dilation, diaphoresis, and tachypnea. With sleep terrors the child cannot be fully aroused or comforted by parental presence. Sleep starts and somniloquy are sleep-wake transition disorders typically occurring during the time of transition from wakefulness to sleeping and vice versa. Sleeps starts are characteristically sudden, muscular jerking of the arms and legs. Somniloquy or sleep talking is common during childhood and associated with sleep terrors. Vocalizations and bodily movements are rare with nightmares

An adolescent presents with a complaint of chest pain. Which associated symptom MOST likely indicates that the underlying problem is of a non-cardiac origin? A. pain with palpation B. nausea C. palpitations D. syncope

The correct answer is A - pain with palpitation Chest pain in the adolescent is a common complaint but is rarely caused by a true cardiac condition. Chest pain is most commonly the result of a musculoskeletal problem that originates in the chest. However, it is important to rule out cardiac causes as this is the primary concern, especially from patients and families. Palpitations, syncope, and loss of consciousness would increase the suspicion of a cardiac origin for the chest pain, while pain that is reproducible with palpation on exam would be more indicative of a costochondritis or musculoskeletal problem

Which laboratory assessment is the BEST indicator of vitamin D deficiency? A. 1,25(OH)2-D (calcitriol) B. 25(OH)-D (cholecalciferol) C. PTH (parathyroid hormone) D. serum calcium

The correct answer is B - 25(OH)-D (cholecalciferol) The best diagnostic study of vitamin D deficiency is the level of 25(OH)-D (cholecalciferol). 1,25(OH)2-D (calcitriol) is the active metabolite of 25(OH)-D, but due to its short half-life it is not a good indicator of vitamin D sufficiency. The pathophysiology of rickets includes: the parathyroid hormone releases calcium from bone; rachitic changes can be seen at the growth plates; and, decreased calcification leads to thickening of the growth plate. Serum calcium and phosphorous are initial screening tests but not the best indicator of vitamin D deficiency.

A mildly ill 4 year old, with a recent positive rapid strep test, returns with abdominal pain and erythematous nonblanching lesions on the legs. What is the MOST likely diagnosis? A. erythema multiforme B. Henoch-Schönlein purpura (HSP) C. immune thrombocytopenia purpura D. scarlet fever

The correct answer is B - Henoch-Schönlein purpura (HSP) Henoch-Schönlein purpura (HSP) is the most common small vessel vasculitis found in young children. The triad of non-thrombocytopenic palpable purpura, colicky abdominal pain, and arthritis is the most common presentation. It appears more commonly in the spring and fall, and is seen more frequently in males ages two to seven. Group A streptococcus is a common trigger for the development of HSP. Gastrointestinal symptoms usually follow the rash and can range from a colicky abdominal pain to profuse bleeding or perforations. When the gastrointestinal symptoms occur first, the diagnosis of HSP can be missed. Signs and symptoms of scarlet fever include sore throat, with erythematous palate and uvula, palatial petechiae, and a fine, papular exanthema which begins in the axilla, groin and neck and spreads within twenty-four hours. These lesions blanch with applied pressure. Erythema multiforme (EM) may present with macules, papules, vesicles or urticarial-like lesions focused primarily on the upper extremities. While the cause of EM is not known, herpes simplex virus is commonly associated with it and Mycoplasma pneumoniae, less frequently. The rash of immune thrombocytopenia purpura is petechial in nature and has a generalized distribution and often associated bleeding from gums and/or mucous membranes.

Which of the following screening criteria would BEST predict dyslipidemia in a child? A. child with blood pressure above 85th percentile for age B. child with body mass index at 90th percentile for age C. parent with high blood pressure at 55 years D. grandparent surviving heart attack at 65 years

The correct answer is B - child with body mass index at 90th percentile for age The increase in childhood obesity has led to an increased interest in lipid screening. It is believed that early detection and treatment of lipid disorders may help prevent adult cardiovascular disease and identify all children with hyperlipidemia. Being overweight, defined as body mass index greater than 85th percentile, is most predictive of dyslipidemia. Parental histories of high blood pressure, high cholesterol, or heart attack are important to note for documentation of family history

Five weeks after joining the cross-country team, an adolescent complains of pain in the front of the left shin that intensifies 15 minutes into each practice run. Plain radiographs of the left leg are normal. Which subsequent test would be MOST appropriate? A. bone scan B. MRI of left leg C. repeat plain films in 1 week D. repeat plain films in 2 weeks

The correct answer is B - MRI of left leg Tibial stress fractures are common in runners. An adolescent who complains of pain in the front of the shin which intensifies 15 minutes into running has symptoms consistent with an anterior tibial stress fracture. It is important to diagnosis this quickly and to make the appropriate orthopedic referral. A bone scan would be positive with this type of stress fracture, but it remains positive for the subsequent 1-2 years after injury. Therefore, it is not useful for assessing healing or ability to return to play. MRI has replaced bone scans as the most sensitive tool for diagnosing stress fractures in long bones. Repeating plain films in 2 weeks can demonstrate periosteal reaction if a stress fracture is present. There is no reason to wait 1-2 weeks for a diagnosis and delay treatment. Treatment may require casting, crutches, or surgery, although the use of pneumatic compression walking boots have shown a reduction in time to return to full activity and pain reduction. Initial treatment includes ice, acetaminophen, and rest, and physical therapy may be ordered. Nonsteroidal anti-inflammatory drugs (NSAIDs) are generally not recommended as they can affect bone healing. During the healing process, cross training activities are generally recommended to include weight lifting, biking, or workouts on an elliptical trainer. Reintroduction of running is gradual and usually based on radiographic determination that the bone has sufficiently healed.

A 17 year old who is approximately 28 weeks pregnant presents with a laceration to her foot after walking barefoot in an animal pen. Her last tetanus immunization (Td) was given at age 11 years. What is the MOST appropriate tetanus prophylaxis? A. dT 0.5 mL IM B. Tdap 0.5 mL IM C. tetanus immune globulin (TIG) 250 units IM D. Tdap administered after 30th week of pregnancy

The correct answer is B - Tdap 0.5 mL IM Because the bacteria Clostridium tetani is ubiquitous in the environment, routine wound management requires evaluation of tetanus vaccine status. Areas of soil where excreta is common are particularly risk prone for the bacteria. Administration of tetanus vaccine depends on the type of wound (clean, minor vs. all other types) and the patient's history of previous tetanus toxoid doses. In 2012, the Advisory Committee on Immunization Practices (ACIP) recommended routine immunization of Tdap during pregnancy for all women. There have been no reported adverse effects for the fetus from Tdap vaccines, and is not believed to increase the risk of adverse events during pregnancy, so one dose is routinely recommended in the last trimester for the purpose of protecting the infant from pertussis in the first few months after birth. In this scenario with a dirty wound at significant risk of contamination, it is acceptable to administer the vaccine at the time of injury, as the vaccine can safely be given at any time during gestation. In general, it is preferred during the last trimester of pregnancy to provide the highest concentration of maternal antibodies transferred closer to birth. DTap is not recommended or approved for pregnant women or any patient >7 years of age.

When counseling parents whose children have had food-induced anaphylaxis, which teaching point is the MOST important? A. Exercise can be a co-factor in food induced anaphylaxis. B. Timely administration of epinephrine is the standard of care. C. Children with severe food-induced allergic reactions have a higher risk for asthma. D. The severity of previous reactions cannot predict the severity of future reactions

The correct answer is B - Timely administration of epinephrine is the standard of care. Parents should be aware of the fact that food-induced allergic reactions can occur during or soon after exercise, and food allergies and asthma often co-exist. Children with asthma tend to have more severe food-induced reactions and the intensity of previous allergic reactions to foods cannot predict the severity of subsequent reactions. Ultimately, in a child with a history of food-induced anaphylaxis, it is MOST important that parents understand that only epinephrine can stop a food-induced allergic reaction and it should be administered immediately when symptoms occur. Reinforcing that epinephrine should always be carried and immediately available is another important teaching point.

Which of the following is considered a strong risk factor for violent behavior in adolescents? A. above average IQ B. antisocial behaviors C. family history of alcohol abuse D. involvement in martial arts classes

The correct answer is B - antisocial behaviors The result of aggressive behavior is violence and injury towards people or property. Violence is a major problem in the U.S. and approximately 60% of children and adolescents have experienced violence in their everyday lives. Violence is an especially strong concern among adolescents. Multiple factors and influences can be attributed to adolescents who commit violent acts including: single parent families, lack of family connectiveness, low IQ, alcohol or drug use, poverty, and poor involvement in the community. Other factors associated with violence include: gang membership, weak social connections or ties, and antisocial behaviors. Antisocial behaviors may include aggression, disregard of authority or the rights of others, deceitfulness, impulsivity, and a lack of remorse. Once an individual engages in acts of violence, the sense of power and control it provides contribute to its addictiveness and habituation. The provider needs to properly assess and educate adolescent patients and their parents or caregivers about the risk of social violence.

Which of the following is the MOST appropriate advice for the proper fitting and use of bike or sports helmets? A. replace helmet every 2 years B. avoid using secondhand helmets C. position helmet high on forehead D. fit helmet to allow some movement side to side

The correct answer is B - avoid using secondhand helmets Helmets should be worn when a child is involved in bicycling, skateboarding or inline skating, when riding all-terrain vehicles (ATVs), motorcycles, or scooters, and when engaging in winter sports such as skiing, ice hockey, or riding snowmobiles. Proper use starts with proper helmet fitting. Guidelines for proper helmet fitting include trying on several sizes and models to find a fit that places helmet low on the forehead, positions the brim so that it is parallel to the ground when the head is upright, securely fastens the chin strap to the point where the helmet will not shift over the eyes, rock side to side, or come off when the child shakes his or her head. Helmets should be replaced every 5 years or sooner, depending on manufacturer recommendations. Helmets involved in a substantial blow should be replaced and secondhand helmets should be avoided. Children are more likely to wear helmets if a parental rule exists about its unconditional use, if parents wear helmets during their own cycling or sports activities, and if there is a mandatory state helmet law.

When prescribing a selective serotonin reuptake inhibitor to treat a child with an anxiety disorder, it is MOST important to A. obtain a complete blood count prior to beginning the medication. B. disclose side effects including the increased risk of suicidal ideation. C. determine the initial dosage based on the severity of the symptoms. D. disclose that this class of medications is not FDA approved for treating anxiety

The correct answer is B - disclose side effects including the increased risk of suicidal ideation. Anxiety disorders in childhood are diagnosed when fears, worries or responses occur outside the range of normal development. Many children have anxiety symptoms manifested as school avoidance. There are many different types of anxiety disorders including generalized anxiety disorder, social phobias, panic disorder, and separation anxiety. There has been widespread research in the area of anxiety disorders showing the benefit of cognitive behavioral therapy (CBT) in addition to pharmacotherapy. Many children experience improvement of anxiety symptoms with CBT alone, while selective serotonin reuptake inhibitors (SSRIs) have been found to be the most effective pharmacological agents. When considering the use of SSRIs in children, the risk and benefits need to be addressed with the parent or caregiver. It is important to disclose all potential adverse effects, including the rare but increased risk for suicial thinking and behavior in some children while taking antidepressant medications. Initial doses should start low with gradual increases based on symptom response and occurrence of adverse effects. It is also important to acknowledge that it can take several weeks to months to achieve a satisfactory, consistent response to the medication.

Caregivers have been administering dextromethorphan before bed to a 3 year old with a three-day history of upper respiratory illness. Which of the following is the BEST advice? A. add a decongestant during the day B. discontinue the antitussive medication C. add an antihistamine to improve symptoms D. continue the dextromethorphan at night

The correct answer is B - discontinue the antitussive medication Coughing is a protective mechanism to clear secretions. Parents and caregivers should be advised that the use of decongestants, antihistamines, and cough medicine is not indicated for children younger than 4 years old, and should be used with caution in children younger than 6 years old. The child should receive symptomatic relief for fever, pain, and nasal congestion using an oral antipyretic and normal saline drops for the nose

Which of the following PRIMARY interventions assists in the prevention of adolescent depression? A. screen for co-morbid conditions B. discuss changes associated with adolescence C. recommend help from professionals rather than friends D. reassure that short-term depression and mood swings can be normal

The correct answer is B - discuss changes associated with adolescence Health practitioners must engage in early interventions to recognize and prevent adolescent depression, despite the fact no one specific program has proved effective. The presence of positive family and peer relationships aid in the identification and resolution of depression in adolescents. Studies have found that depression is not part of any normal adolescent behavior. In help seeking behaviors, many adolescents feel most comfortable reaching out to a friend or someone who cares about them for validation of their feelings and this should be supported. Adolescents should be screened for co-morbid conditions, but this is a secondary intervention. Education is the primary issue in prevention of depression. Children should be educated regarding the changes of adolescence as they approach this stage and parents and educators should have education to recognize depression early

Recommendations for the use of supplemental iron in otherwise healthy infants and children includes that A. preterm infants fed human milk should receive supplementation at 2 months of age. B. exclusively breastfed infants should begin supplementation at 4 months of age. C. exclusively breastfed infants should begin supplementation at 6 months of age. D. toddlers should receive supplementation with liquid or chewable iron products

The correct answer is B - exclusively breastfed infants should begin supplementation at 4 months of age. Research in the past few years has supported the concern that iron deficiency anemia (IDA) and iron deficiency without anemia can have long lasting detrimental effects on a child's neurodevelopment. For example, adequate iron stores are associated with higher Bayley developmental scores and better visual acuity. Infants born prematurely are deficient in total body iron, but term infants have enough iron stores for the first 4 - 6 months of life. After being born with more than adequate hemoglobin levels, full term infants will experience a physiologic decline in hemoglobin and total blood volume in the first few months of life. Other factors affect fetal iron stores, including maternal conditions such as diabetes or hypertension with intrauterine growth retardation. Despite a previous assumption that the small amount of iron in breast milk was adequate for breastfed infants, it is now recommended that exclusively breastfed term infants receive iron supplementation of 1 mg/kg/day, starting at 4 months of age and continuing until iron-containing foods are introduced. Preterm infants who are fed human milk should receive an iron supplement of 2 mg/kg/day by 1 month of age. Iron for toddlers is best delivered by a diet rich in red meats, fortified cereals, vegetables with iron, and fruits with vitamin C. In this age group, if the recommended 7 mg/day is not accomplished by diet, then liquid or chewable supplements should be offered.

A 5 year old complains of a painful left eye after being accidentally scratched by a sibling two hours ago. Fluorescein exam shows a small central corneal abrasion. The MOST appropriate management during the first 24 hours is A. frequent application of topical anesthetics. B. frequent application of topical antibiotic. C. occlusive patching of the injured eye. D. monitoring with return office visit.

The correct answer is B - frequent application of topical antibiotic. Accidental abrasion of the corneal epithelium is a common eye injury in children causing pain, tearing, and photophobia. An abrasion can be detected by examining the eye with a Wood's lamp after instillation of fluorescein dye. The one time use of a topical ophthalmic anesthetic may be useful in gaining cooperation for an adequate eye exam. The goal of treatment is rapid healing of the abrasion. Until such healing occurs, the eye should be protected from infection by the use of a topical ophthalmic antibiotic applied 4 times a day for a few days. If there is a positive response to treatment, drops continue for 2-3 days. The repeated use of a topical anesthetic is not recommended, as these medications can cause corneal toxicity and inhibit the blinking reflex. Topical steroids are not recommended as they lower the eye's resistance to infection. Oral acetaminophen or ibuprofen and intermittent cool compresses may help to manage discomfort. The use of topical nonsteroidal anti-inflammatory drops is being studied in the treatment of some sterile corneal abrasions, such as those acquired during laser treatment of refractive errors in adults, but are not recommended in management of traumatic corneal abrasions in children. Patching is no longer recommended for most corneal abrasions, as it does not reduce discomfort or speed healing and makes instillation of antibiotic medication more difficult. Most corneal abrasions heal steadily over the first 24-48 hours. Persistent or increasing pain or discomfort after the first 24 hours indicates the need to refer to ophthalmology for further evaluation.

An infant with a history of prematurity presents with a full anterior fontanelle, and slightly separated sagittal suture. Head circumference has increased from 50th to the 90th percentile on the growth chart. Which of the following is the MOST likely diagnosis? A. craniosynostosis B. hydrocephalus C. intraventricular hemorrhage D. plagiocephaly

The correct answer is B - hydrocephalus Hydrocephalus is the abnormal accumulation of cerebrospinal fluid (CSF) within the ventricles in the brain, which, if untreated, leads to increased intracranial pressure. Symptoms in infants include increasing head circumference growth velocity, full or bulging fontanelles, separated cranial sutures, poor feeding, lethargy, a plateau in achieving developmental milestones, and irritability. Hydrocephalus is considered a neurosurgical emergency and requires prompt referral. Craniosynostosis is a birth defect in which one or more of the cranial sutures close permanently before the infant's brain is fully formed. The brain continues to grow giving the infant's head a misshapen appearance. It usually involves a single suture. Rarely, it is caused by a genetic syndrome. Treatment involves surgery to correct the shape of the head and allow for normal brain growth. Early diagnosis and treatment are important to allow the brain adequate space to grow and develop. Plagiocephaly develops when outside forces place pressure on the cranium as it grows, causing flattened areas, which commonly occurs when infants regularly sleep in one position. Plagiocephaly occurs more often in premature infants as their skulls are especially pliable. Treatment includes special exercises, repositioning the infant's head from left to right/right to left while sleeping supine, varying the sleep position, practicing more tummy time, and wearing corrective headbands or using molding cups. Intraventricular hemorrhage (IVH) is bleeding into the brain's ventricular system, where cerebrospinal fluid is produced and circulates through the subarachnoid space. It can result from physical trauma or from hemorrhaging with a stroke. It is common among premature infants, especially those weighing less than 1500 grams. IVH usually occurs within first 72 hours of life for premature infants. Symptoms include seizure, apnea, bradycardia, and bulging of fontanels

A 10 year old presents with a 6 month history of headaches occurring twice per month described as frontal, pounding, lasting for most of the day, and accompanied by nausea and light sensitivity. Which is the BEST initial treatment? A. amitriptyline (Elavil) B. ibuprofen (Advil) C. promethazine (Phenergan) D. sumatriptan (Imitrex)

The correct answer is B - ibuprofen (Advil) Migraine without aura is the most common form of migraine headache in children. It is characterized as an intense frontal or temporal headache, lasting from 1 to 48 hours, accompanied by nausea, vomiting, and sensitivity to light and sound. NSAIDS are the first line treatment for acute migraines in children. Acetaminophen has a faster onset of action than ibuprofen, but ibuprofen has shown greater headache resolution. Many newer medications for migraines have not been adequately tested in children. Sumatriptan is approved for use in adolescents. Daily antidepressants such as amitriptyline are widely used for preventive treatment in migraine management. Amitriptyline has been found to be 50-80% efficacious in children, but needs to be used with caution in children younger than 12 years of age. It is not the first-line therapy. Promethazine has indications as an adjunct medication for postoperative nausea and vomiting in children older than 2 years

When providing anticipatory guidance to parents whose infant has colic, which information is MOST appropriate to include? A. fever is often an accompanying symptom of colic B. infants with colic are hypersensitive to stimuli C. teething can stimulate a colicky response D. symptoms increase at 3 - 4 months of age

The correct answer is B - infants with colic are hypersensitive to stimuli An infant's prolonged bouts of crying are stressful to parents and providers. The criteria for colic include: • crying that persists 3 or more hours a day with episodes occurring 3 or more days per week, and • lacking any apparent reason or cause in an otherwise health infant. It is imperative to rule out significant pathology for an infant in distress who has no way to communicate feelings but through crying. The differential diagnosis list is long and must be considered in an organized way. First, determine if there is fever to indicate infection as it is not a usual symptom of colic. Teething usually does not occur until close to six months of age, which is after the typical period of colic, between 1 week and 2 - 3 months of age. Physical examination of the infant should reveal normal growth and development, and helps to differentiate neurological, metabolic, allergic, or traumatic causes for the crying. Once pathological causes for the crying have been ruled out, parents need reassurance that the crying is the only way the infant can communicate not feeling well. Infants with colic are frequently hypersensitive to their new extrauterine environment. Review findings with parents, educating them on the colicky baby syndrome, and what to look for when crying is not associated with infantile colic

An adolescent presents with an infected pierced naval, revealing moderate swelling, erythema, and exudate. The piercing, kept open with a gold ring, is one month old, and has been maintained with antibacterial soap and water cleansing. The adolescent reports swimming in a lake one week ago. Which of the following is the MOST appropriate? A. remove jewelry from naval B. initiate oral antibiotics C. apply topical barrier cream D. cleanse with hydrogen peroxide

The correct answer is B - initiate oral antibiotics Infections are one of the most common complications following body piercing. Each piercing site has a typical length of time for healing; the naval area takes 2-12 months to heal after piercing. While there is no uniform treatment of infected piercing sites, a course of antibiotic therapy which is effective against Staphylococcus aureus is advised, in addition to cleansing and topical antibiotic. Decisions to remove the jewelry should be made only if it would prevent healing, be a continued source of infection, or if removal would facilitate drainage; otherwise, it is better to leave the jewelry in place. If systemic symptoms occur, the adolescent should be admitted to the hospital for incision and drainage of the site and intravenous antibiotics

An infant born at 35 weeks has which of the following risks? A. excessive wakefulness B. poor sucking strength C. hyperglycemia D. colic

The correct answer is B - poor sucking strength In the past decade or more, much has been learned about late preterm infants, those not markedly premature and yet not full term. The term "late preterm birth" or "late preterm infant" includes infants born between 34 weeks and 36 6/7 weeks. This late preterm infant has few wakeful periods so time awake should be used for feeding. These infants can also have significant difficulty in developing a coordinated suck, swallow, and breathing pattern. At birth, the late preterm infant has an increased risk of developing apnea, bradycardia, hypoglycemia, hyperbilirubinemia, hypothermia, airway instability, excessive lethargy, and sepsis. Infants in this gestational age group are not more likely to develop colic than fullterm infants

During the winter, a school-age child with a history of asthma presents with a one-day history of rhinorrhea, sore throat, dry hacking cough, a few episodes of vomiting and general malaise. Which is the most appropriate INITIAL diagnostic test? A. complete blood count B. rapid influenza test C. rapid respiratory syncytial virus antigen D. rapid strep test

The correct answer is B - rapid influenza test Influenza typically presents with fever, headache, malaise, myalgias, nonproductive cough, sore throat, nasal congestion, rhinitis, and less commonly, abdominal pain, nausea, vomiting, and diarrhea. Fever, sore throat, abdominal pain and vomiting (occasionally), are symptoms associated with strep throat, a bacterial infection. Stuffy and runny nose, and cough are usually seen with colds and respiratory viruses. A complete blood count can be helpful in determining viral versus bacterial etiology of infection but is not the most appropriate initial test. Antiviral medications can be used to reduce duration and severity of influenza and decrease the risk of complications if started within 48 hours of symptom onset. Since influenza can be especially dangerous for children with asthma, prompt recognition and consideration of antiviral treatment is essential. Respiratory syncytial virus (RSV) also causes signs of upper respiratory infection but is more often associated with low-grade fever and is not associated with vomiting. The highest incidence of RSV is between 6 weeks and 7 months of age, with maternal serum antibodies providing protection before 6 weeks of age except among infants born preterm. Bronchiolitis occurs most commonly among infants infected before 1 year of age. The majority of children are infected with RSV by 2 years of age. Reinfection occurs throughout childhood with the severity of illness usually lessening with subsequent infection, related in part to partial acquired immunity. Treatment of RSV is supportive. Immunoprophylaxsis is available with the administration of palivizumab, with the American Academy of Pediatrics (AAP) providing recommendations for its use among infants born preterm and for other infants and children, such as those who are immunocompromised or with chronic pulmonary disease.

For a 21 month old with persistent fever following 48 hours of antibiotics for a first febrile urinary tract infection, which of the following is the MOST appropriate diagnostic test? A. repeat urinalysis B. renal ultrasound C. voiding cystourethrogram D. dimercaptosuccinic acid scan

The correct answer is B - renal ultrasound The 2011 American Academy of Pediatrics Clinical Practice Guideline: The Diagnosis and Management of the Initial Urinary Tract Infection in Febrile Infants and Young Children 2-24 Months of Age was reaffirmed in November 2016. This clinical practice guideline defines urinary tract infection (UTI) in 2 to 24 month olds as the presence of pyuria and greater than 50,000 colonies in a urine sample obtained by catheterization or suprapubic aspiration. Renal ultrasounds are obtained for this population during the acute illness if response to treatment is not following the expected course. Obtaining the renal ultrasound during the acute illness can identify renal and perirenal abscesses which could interfere with clinical improvement. Dimercaptosuccinic acid scans are a tool used in research to identify pyelonephritis. A voiding cystourethrogram (VCUG) is not necessary after the first febrile UTI unless the findings on the renal ultrasound indicate abnormalities such as high grade reflux, scarring or obstruction. While a urinalysis will identify the presence or absence of pyuria, this information is not helpful in determining the cause of persistent fever during antibiotic therapy.

A 12 month old with no recent medication history presents with fever, malaise, bullous impetigo, and diffuse erythroderma in flexural and periorificial areas. Mucous membranes are not affected, but crusting and fissuring is evident around the eyes, mouth and nose. When the skin is gently rubbed, the epidermis in the area is denuded. These findings are characteristic of: A. Kawasaki disease B. staphylococcal scalded skin syndrome C. Stevens-Johnson syndrome D. toxic epidermal necrolysis

The correct answer is B - staphylococcal scalded skin syndrome Staphylococcal scalded skin syndrome (SSSS) is caused by a toxin released by the bacteria Staphylococcus aureus. A common finding with SSSS is a positive Nikolsky's sign, such that the outer epidermis separates from the basal layer with gentle friction. SSSS may be differentiated from other conditions by the lack of history for medication exposure, mucous membrane involvement, or respiratory symptoms. Toxic epidermal necrolysis is caused by recent medication use and presents with fever, sore throat, malaise, and mucous membrane involvement with generalized epidermal sloughing. Stevens-Johnson syndrome is caused by medications or a viral syndrome. It begins with high fever, cough, sore throat, vomiting, diarrhea, chest pain and arthralgia. Symptoms can last 1-14 days followed by erythematous macules on the head and neck, which can spread to the trunk and extremities. Mucous membranes are almost always involved including the eyes, nose and mouth. Kawasaki disease presents with sudden systemic vasculitis throughout the body. Clinical evidence supports an infectious cause and it is more common in late winter/early spring. The child most often has had multiple days of fever and is irritable with nonpurulent conjunctivitis, dry cracked lips, a strawberry tongue, swollen cervical lymph nodes, polymorphous rash, tachycardia and swollen extremities.

A 6 year old with viral induced asthma presents with 3 days of low grade fever, rhinorrhea, and cough. In addition to wheezing, which of the following findings are MOST likely noted on physical examination? A. tachypnea and crackles to bases B. tachypnea and intercostal retractions C. intercostal retractions and crackles to bases D. productive cough and prolonged inspiration

The correct answer is B - tachypnea and intercostal retractions Asthma is the most common chronic respiratory disease in children and is the number one reason for emergency visits. It is characterized by chronic airway inflammation and hyperresponsiveness to triggers such as viral illness, environmental irritants, and exercise. Hallmark clinical findings are due to bronchoconstriction and airway obstruction with mucus and other inflammatory byproducts. The most classic finding in asthma is wheezing. A prolonged expiratory phase is common due to distal air trapping along with mucus plugging from inflammatory mediators. Accessory muscles are recruited to help ventilate against the resistance of constricted and obstructed airways. Intercostal retractions are often seen in mild asthma exacerbations. As the obstruction becomes more severe suprasternal retractions, tracheal tug, and nasal flaring may occur. A dry protracted cough would be expected. Prolonged inspirations would not be typical or expected. Wet productive cough and crackles to lung bases would also not be expected and would be more consistent with a pneumonia and resulting fluid infiltration versus an airway obstruction.

The child at highest risk for having an elevated blood lead level is a: A. 3 month old exclusively breastfed infant B. 6 month old who lives in a home built after 1970 C. 2 year old with iron deficiency anemia D. 2 year old who is a picky eater

The correct answer is C - 2 year old with iron deficiency anemia The amount of lead absorbed from the gut is increased in children with nutritional deficiencies such as iron deficiency anemia (IDA). Iron deficiency anemia is often a comorbidity of lead poisoning. The hand-to- mouth behavior of infants and young children increases their risk for lead exposure. However, living in a home built after 1970 reduces the risk since residential paint used in that era should not have been lead based. Infants more than 4 months of age exclusively breast fed without supplemental iron are at increased risk of IDA. A child who is a picky eater may or may not be at high risk for IDA, depending on foods actually eaten.

For acute bacterial sinusitis, the optimal duration of antimicrobial therapy is: A. 7 days total treatment B. 14 days total treatment C. 7 days after symptoms have resolved D. 10 days after symptoms have resolved

The correct answer is C - 7 days after symptoms have resolved Acute bacterial sinusitis is diagnosed when a child presents with an upper respiratory infection (URI) with persistent illness described as daytime cough and nasal discharge lasting more than 10 days. It can also be diagnosed when a child with an improving URI suddenly gets worse with fever, daytime cough or new nasal discharge. Systematic study of the optimal duration of antibiotic treatment for acute bacterial sinusitis is lacking. Recommendations ranging from 10 to 28 days of treatment have been made. The American Academy of Pediatrics Clinical practice guideline for the diagnosis and management of acute bacterial sinusitis in children aged 1 to 18 years suggests continuing treatment for 7 days after resolution of signs and symptoms. This approach allows the course of therapy to be tailored to the individual, with a minimum course of 10 days. Further, this avoids prolonged treatment in those who are asymptomatic and less likely to complete a longer course.

The parents of a 2 year old are concerned that their child is having temper tantrums in public settings. Which is the BEST response? A. Modifying expectations when in a public place is acceptable to promote calming. B. The temper tantrum is an indication that the child is tired and needs a nap. C. Address any safety concerns while ignoring the child's display of behavior. D. Respond to the tantrum with firm expectations for public behavior

The correct answer is C - Address any safety concerns while ignoring the child's display of behavior. Temper tantrums are common in children with a peak age of 18-28 months and may occur as often as once a week in this age group. A temper tantrum is often a sign that a child is angry or frustrated and is trying to achieve autonomy. Appropriate responses and interventions by the parents can assist the child in attaining developmental mastery. Some suggestions include trying to prevent a tantrum by offering the child achievable choices, maintaining an environment that provides positive reinforcement for desired behavior, and fighting only those battles which need to be won. If a child does exhibit a tantrum, provide a safe environment and do not over-react to the behavior. Stay nearby the child during the temper tantrum, avoid eye and physical contact, and provide positive reinforcement when the behavior improves. Parents should be advised following the tantrum to reassure the child that the frustration is understandable but that the behavior is not. By punitively responding to the child's behavior, this can actually reinforce the opposition in the future. The child may need to be restrained if in danger from the environment, so holding the child may be appropriate. Giving in to the child reinforces the behavior, despite where the tantrum occurs

In a 2 month old with visible rib fractures on radiograph, the NEXT most critical evaluation to obtain is a: A. long bone series B. coagulation profile C. CT scan of the head D. retinal ophthalmologic exam

The correct answer is C - CT scan of the head Posterior rib fractures associated with accidental trauma are rare. Posterior fractures can be seen in infants who have been shaken as the perpetrator hands are typically wrapped around the infant's thorax during the shaking, with the vertebrae acting as a fulcrum. These findings should alert the provider to consider abusive head trauma (AHT) (formerly known as shaken baby syndrome). Subdural and subarachnoid hemorrhages are the most common acute intracranial injuries seen in AHT and are associated with high rates of morbidity and mortality. Thus, the most important study to do next is a CT scan. Studies have shown that nearly one third of confirmed abusive head trauma cases were missed on initial presentation, and many infants then sustain additional brain injury along with poorer neurologic outcomes because of the delay in diagnosis. Long bone studies will be needed as part of a thorough work-up of non-accidental trauma, but the skull would be the most critical area to image first. Coagulation studies are done to rule out any coagulation problem associated with injury to the brain and are important for medico-legal reasons, but again, brain studies take precedence. A thorough ophthalmologic exam is needed in suspected cases of AHT—preferably done by a pediatric ophthalmologist

A 14 year old and her parent express concern that she has not begun to have her menstrual period. Pubic hair development started about 2 years ago, and breast development began about 18 months ago. She has grown 2 inches in height in the past 6 months. Which would be the MOST accurate response? A. Menses generally begins in females within 1 year after breast development begins. B. Because menses may not begin until the age of 18, evaluation is not needed at this time. C. Menses will probably begin in the next 6 - 12 months, so a wait and see approach is appropriate. D. Appearance of pubic hair before any breast development indicates the need for endocrine evaluation

The correct answer is C - Menses will probably begin in the next 6 - 12 months, so a wait and see approach is appropriate. Adolescents and their parents often bring questions and concerns about pubertal development to their health care providers. Monitoring the progression of puberty requires knowledge of Tanner staging and the influence of hormones produced in the central nervous system (CNS), adrenal glands, and ovaries or testicles. In 70-75% of females, breast development signals the beginning of puberty. In the remaining 25- 30% of females, the appearance of pubic hair is the first sign of pubertal development. Breast development (thelarche) and pubic hair development (adrenarche) are controlled by different hormones and staged separately. Breast development is controlled primarily by ovarian estrogens, and pubic hair development is regulated by adrenal hormones DHEA, DHEAS, and androstenedione. The female adolescent growth spurt height velocity of approximately 3.3 inches of growth per year occurs during Tanner stages 2 and 3 . Menarche typically occurs 2 - 2.5 years after breast development begins. By the time menarche occurs, the height growth velocity curve is declining. The criterion for diagnosing delayed puberty in females includes no pubertal development by age 13, pubertal maturation not complete within 4 years, or no menses by age 16.

In addition to use of the supine position, which of the following is a sudden infant death syndrome risk reduction strategy to teach parents? A. use a humidifier in the room to avoid overheating B. allow co-sleeping to monitor infant sleep position C. give the infant a pacifier at bed and nap time D. use a device to maintain supine sleep position

The correct answer is C - give the infant a pacifier at bed and nap time Rates of sudden infant death syndrome (SIDS) have declined more than 50% in the U.S. and around the world over the past 10 years or more in part, because of national education campaigns. Risks for SIDS include genetic risk factors, non-modifiable factors, and modifiable factors such as cigarette smoking, drug and alcohol use, and infant care practices. Recommendations for pacifier use have been found to reduce the risk of SIDS. The mechanism of action is unknown but it is thought that pacifier use may alter arousal thresholds or autonomic responses during sleep. There is no recommendation for or against humidifier use as a risk reduction strategy. It is recommended that infants sleep in the same room as their parents but on a firm mattress in their own crib or bassinette. Devices advertised to maintain sleep position or reduce the risk of rebreathing are not recommended

Which of the following children should receive immunization with 23-valent pneumococcal conjugate vaccine (PPSV23)? A. a healthy 2 year old who has not received any doses of PCV 13 B. a healthy 2 year old who has received 3 doses of PCV13 C. a 4 year old with sickle cell disease who received PCV 13 series with last dose at age 15 months D. a 5 year old with intermittent asthma controlled with albuterol who received 4 doses of PCV 13

The correct answer is C - a 4 year old with sickle cell disease who received PCV 13 series with last dose at age 15 months Due to ongoing research, the vaccine to prevent diseases caused by streptococcus pneumoniae has changed with increasing coverage for multiple serotypes of the bacteria. According to the 2018 immunization schedules published by the Centers for Disease Control, the pneumococcal conjugate vaccine 13-valent (PCV13) should be administered in a 4 dose series to all children with the last dose given between ages 12 and 15 months of age. Healthy children 24-59 months of age who have not received PCV13, should receive one dose. All recommended PCV13 doses should be administered prior to PPSV23 vaccination if possible. Children ages 2 - 5 who have high risk conditions should receive the PPSV 23 beginning at age 2 years. Types of chronic illness that apply include chronic heart disease, asthma (if treated with high-dose oral corticosteroid therapy), diabetes mellitus, cochlear implant, sickle cell disease and other hemoglobinopathies, anatomic or functional asplenia, HIV infection, chronic renal failure, diseases associated with treatment with immunosuppressive drugs or radiation therapy, solid organ transplantation, or congenital immunodeficiency.

First-line therapy for a 5 year old with acute otitis media and concurrent purulent conjunctivitis is: A. amoxicillin 90 mg/kg/day B. azithromycin 10 mg/kg/day on day 1, then 5 mg/kg/day on days 2-5 C. amoxicillin/clavulanate (Augmentin) with 90 mg/kg/day of amoxicillin component D. cephalexin (Keflex) 50mg/kg/day

The correct answer is C - amoxicillin/clavulanate (Augmentin) with 90 mg/kg/day of amoxicillin component To provide additional -lactamase coverage, amoxicillin/clavulanate (Augmentin) is recommended as initial therapy for acute otitis media (AOM) in children who have received amoxicillin in the previous 30 days, those with concurrent purulent conjunctivitis, or those who have a history of recurrent AOM unresponsive to amoxicillin. Therapy should be initiated with high-dose amoxicillin-clavulanate (90 mg/kg/day of amoxicillin, with 6.4 mg/kg/day of clavulanate). Alternative initial antibiotics include: • cefdinir (14 mg/kg per day in 1 or 2 doses), • cefuroxime (30 mg/kg per day in 2 divided doses), • cefpodoxime (10 mg/kg per day in 2 divided doses), or; • ceftriaxone (50 mg/kg, administered intramuscularly). The optimal duration of therapy for AOM is unclear. A 10-day course was extrapolated from treatment of streptococcal pharyngotonsillitis. There is some data that favors a 10-day course over shorter courses in children under 2 years of age; therefore, a standard 10-day course is recommended in children under 2 years of age. According to the American Academy of Pediatrics Clinical Practice Guideline: The diagnosis and management of acute otitis media, in children 2 to 5 years of age with mild or moderate AOM, a 7-day course of oral antibiotics is equally effective. A 5- to 7-day course of treatment is adequate for children with mild to moderate AOM who are 6 years of age or older

An adolescent mentions that she received her driver's license two weeks ago and will now be picking up her sibling from preschool. Which of the following is appropriate anticipatory guidance? A. reduce speed when driving after 10 pm B. pay close attention to the sibling while in the car C. drive with no more than one adolescent passenger D. place sibling in middle backseat with seat belt securely across thighs and chest

The correct answer is C - drive with no more than one adolescent passenger Risk for motor vehicle accident (MVA) is highest in the first year a teen has their license, making driving safety an important anticipatory guidance topic. Crash risk goes up when teens drive with other teens in the car and the CDC recommends limiting the number of teen passengers to zero or one. For all ages, fatal crashes are more likely to occur at night, but the risk is greater for adolescents. The CDC recommends that adolescents should be off the road by 9-10 pm at least for the first 6 months of driving. CDC passenger restraint guidelines state that children between the ages of 2 and 5 should be in a forward-facing car seat or booster seat, depending on height and weight, and should not be allowed to use a seat belt alone. Distractions increase the risk of being in a crash so the adolescent should not be advised to pay close attention to the sibling while driving

A caregiver is concerned about her 14 month old's in-toeing and frequent falls when walking. Physical exam reveals a wide-based externally rotated gait, with a foot progression ankle of minus 5 degrees. The MOST appropriate response is to A. refer to an orthopedist for surgical intervention. B. refer to physical therapy for medial tibial torsion. C. explain that in-toeing typically self resolves by 8 years of age. D. obtain radiographs of pelvis and feet for potential hip dysplasia

The correct answer is C - explain that in-toeing typically self resolves by 8 years of age. The most common cause of in-toeing in young children is internal femoral torsion (femoral anteversion), a relatively benign condition that typically self-resolves by 8 years of age. Appropriate management includes observation, and non-urgent referral to orthopedics is warranted if there is >70 degree medial or <10 degree lateral hip rotation. Metatarsus adductus causes in-toeing with a foot progression angle of >15 degrees. Metatarsus adductus with a flexible foot is typically responsive to physical therapy; however, a non-flexible foot warrants referral to orthopedics. Untreated hip dysplasia in an ambulatory child presents with toe-walking, leg-length discrepancy, and painless limping or waddling gait

The MOST common symptom of hepatitis A in a 14 month old is: A. ascites B. dark urine C. fever D. jaundice

The correct answer is C - fever Hepatitis A is a viral infection acquired through contaminated food and water, and person to person contact, as well as poor hand hygiene via the fecal-oral route. Hepatitis A is a significant public health concern. Young children with hepatitis A tend to be asymptomatic or have only mild flu-like symptoms including fever, nausea, and anorexia. Few children younger than 6 years have jaundice. Older children usually are symptomatic, with symptoms typically lasting less than 2 months and jaundice occurring in 70% of older children and adolescents. A symptom of jaundice is dark urine. The elderly have more disease complications with hepatitis A including ascites, prolonged cholestasis, pancreatitis, and death from liver failure.

A 10 month old has had apparent abdominal pain for the past 4 hours. During episodes of pain, the infant screams inconsolably and draws his legs towards the abdomen, has vomited twice, but has had no diarrhea. Between episodes the infant is calm and quiet. The MOST likely diagnosis is: A. appendicitis B. acute gastroenteritis C. intussusception D. Meckel's diverticulum

The correct answer is C - intussusception Acute onset of abdominal pain requires immediate investigation. The provider must be able to distinguish between conditions that can safely be managed at home and those requiring immediate referral. Meckel's diverticulum usually involves painless rectal bleeding. Acute appendicitis can occur in infants and must be considered in the differential diagnosis. The pain history of acute appendicitis is usually more continuous. Acute gastroenteritis may involve intermittent pain and cramping, but more typically the child appears ill, and vomiting and diarrhea play a more prominent role. The pain of intussusception is very severe, with periods of well-being in between. There is frequently vomiting, however stools initially are normal. Later stools can be like red jelly in appearance. A sausage shaped mass is frequently palpable in the right upper quadrant and aids in diagnosis

A school-age child with severe atopic dermatitis is prescribed tacrolimus (Protopic) ointment. Potential adverse effects to educate caregivers about include: A. cutaneous atrophy B. hypopigmentation C. local burning D. telangiectasia

The correct answer is C - local burning Topical steroids have been used for treatment of atopic dermatitis (AD) and remain the first-line treatment. Two immunomodulating agents, tacrolimus (Protopic) and pimecrolimus (Elidel), have been approved as second-line agents for treatment of moderate-to-severe AD in children 2 years of age and older. These products inhibit T-cell function and have proven effective in reducing both the inflammation and pruritus of AD without the long-term dermatologic changes that topical steroids can cause. The most common adverse effects of tacrolimus are local burning or stinging sensations. While common, these effects tend to lessen after several applications. Educating patients and families on this can help avoid early and perhaps unnecessary discontinuation of treatment. During times of severe skin disease, the risk of secondary infections is increased. Tacrolimus is not recommended for use during an acute skin infection. There is data that topical tacrolimus used on non-infected lesions has been associated with decreased colonization with Staphylococcus aureus; this is thought to be related to reduced inflammation and improved barrier function of the skin. There is a U.S. boxed warning on both tacrolimus and pimecrolimus related to an association with rare cases of malignancy. Short-term and intermittent use is recommended. For tacrolimus specifically, it is recommended that children 2 through 15 years of age should use only the lower concentration of 0.03% ointment. The warning is based on systemic exposure of laboratory animals and rare cases of adults who used these medications and subsequently developed lymphoma and skin cancers. Further, widespread off-label use in children under 2 years of age has raised concern. No reports of cancers in children have been made in over 15 years of market availabililty. Ongoing 10 year surveillance studies to address these concerns have not found malignancy rates different from those in the general pediatric population

Which of the following lab abnormalities is MOST suggestive of iron-deficiency anemia? A. low hemoglobin B. low mean corpuscular volume C. low serum ferritin D. low serum iron

The correct answer is C - low serum ferritin Iron deficiency anemia is the most common nutritional deficiency in the world. Hemoglobin and hematocrit are used to screen for iron deficiency and are not abnormal until frank anemia has presented. Serum ferritin is the most sensitive indicator for iron deficiency. As iron stores decrease, iron becomes unavailable. With less available hemoglobin in each cell the red cells become smaller. This is best quantified by the decrease in mean corpuscular volume. Serum iron levels vary greatly during the day and with food intake. Serum iron, serum transferrin, total iron-binding capacity (TIBC), and unbound ironbinding capacity (UIBC) reflect circulating iron

Which of the following has been identified as a potential cause of bowel perforation if ingested? A. jewelry beads B. latex balloons C. multiple magnets D. marbles

The correct answer is C - multiple magnets Ingestion of magnets poses a danger to children, and the number of magnets is thought to be critical. If a single magnet is ingested there is less likelihood of complications. If more than 2 magnets are ingested the magnetic poles are attracted to each other across the bowel wall which cause obstruction, leading to necrosis and intestinal perforation. Latex balloons, marbles and jewelry beads pose choking risks to children but do not carry the risk of bowel perforation. These items are most often successfully passed in the stool if they advance beyond the upper airway and esophagus

A 6 month old has an excoriated, erythematous rash in the diaper area with pinpoint satellite lesions. Which topical treatment would be MOST appropriate? A. 1% hydrocortisone cream B. mupirocin ointment C. nystatin ointment D. tacrolimus cream

The correct answer is C - nystatin ointment Diaper rash consisting of sharply defined erythematous patches with satellite lesions or pustules is consistent with Candida albicans diaper dermatitis. This is best treated with an antifungal topical agent such as nystatin, miconazole, clotrimazole, or ketoconazole. Hydrocortisone may assist in decreasing the inflammation, but will not adequately treat the underlying cause. Mupirocin would be ineffective since it is an antibacterial agent. Tacrolimus is limited to use as a second-line agent for short term and intermittent treatment of moderate to severe atopic dermatitis for non-immunocompromised patients

A child with no symptoms of hyperkalemia has a potassium level of 5.8 mEq/L reported on a basic metabolic panel. Which is the MOST appropriate next step? A. obtain a capillary specimen B. decrease potassium in the diet C. obtain a free flowing venous specimen D. discontinue any use of nonsteroidal anti-inflammatory drugs

The correct answer is C - obtain a free flowing venous specimen The most common cause of hyperkalemia in infants and children is pseudo-hyperkalemia, which refers to an elevated serum potassium concentration on a laboratory test when the actual plasma potassium concentration in the blood within the body is normal. This false elevation in the potassium level of a sample may arise from hemolysis related to tourniquet application, traumatic venipuncture or probing, small catheter diameters, excessive force on the syringe either during specimen collection or transfer, and other mechanical factors. This may also occur secondary to hematologic abnormalities including leukocytosis or thrombocytosis. Samples obtained via heelstick in young infants are particularly susceptible to hemolysis. When pseudo-hyperkalemia is suspected, such as in the case of a child without symptoms or risk factors for hyperkalemia, a new specimen should be obtained from a free-flowing venous sample and the test should be repeated before any treatment is administered. True hyperkalemia is typically associated with: inadequate excretion of potassium associated with renal dysfunction, extensive traumatic injuries, or in the presence of significant acidosis; or with increased exogenous intake of potassium either through diet or supplementation, blood transfusions, poisoning or gastrointestinal bleeding. A number of medications including nonsteroidal anti-inflammatory agents, potassium-sparing diuretics, and trimethoprim can increase serum potassium levels. Dietary potassium is unlikely to cause significant increase in serum potassium levels.

Disclosure of which of the following during a HEEADSSS assessment is a PRIORITY for further discussion? A. having sex with both males and females and consistently using condoms B. recently ending a significant relationship after several months together C. participating in social media cyberbullying of a classmate D. attending family therapy related to parent's divorce

The correct answer is C - participating in social media cyberbullying of a classmate The HEEADSSS assessment is a screening interview allowing the provider to take a rapid psychosocial history of an adolescent to identify potential areas of concern. HEEADSSS stands for home, education/employment, eating, activities, drugs and alcohol, sexuality, suicide/depression, and safety. Sexual health and relationships are topics that may warrant further discussion, but witnessing or participating in cyberbullying on social media about a classmate is the priority. Cyberbullying is use of the internet, social media, text messages, or other digital technology to bully others. If a child discloses that they are being bullied or witnessing bullying the health care provider should ask the child or adolescent to provide more information about what is happening. Bullying can have significant negative emotional, developmental, or behavioral effects on children who bully, who are bullied, and who witness bullying. Cyberbullying has also been directly linked to suicide attempts.

A 16 year old presents with a three-hour history of acute pain and swelling of the scrotum, severe enough to make examination difficult. What should be the FIRST course of action? A. Doppler study of the scrotum B. transillumination of the scrotum C. refer to the emergency department D. obtain history of sexually transmitted infections

The correct answer is C - refer to the emergency department Testicular torsion is a surgical emergency with ischemic injury, abscess, atrophy, and infertility occurring if reduction is not preformed within 6-12 hours. If surgery is not performed within specified time parameters, the testicle may have to be removed. Transillumination of the scrotum is used to evaluate for hydrocele, not testicular torsion. A Doppler study is useful to show blood flow to the area, but delays diagnosis. A detailed history is useful, but should not delay referral to the emergency department and surgery

The caregivers of an 18 month old are both overweight. Appropriate guidance to address their child maintaining a healthy weight should include A. limiting carbohydrate intake. B. offering vitamin supplementation. C. responding to child's cues for hunger and satiety. D. returning for weight and BMI measurements every 6 months

The correct answer is C - responding to child's cues for hunger and satiety. Caregivers of toddlers should allow their child to initiate and guide feeding. They should respond to cues of both hunger and satiety and not adopt a "clean everything on your plate" approach. Many foods are fortified and supplements should not be a substitute for food. Additional vitamin supplementation at this age may risk excess vitamin intake. Growing muscle mass and growth in general require a high protein intake. BMI measurement is done for children greater than 2 years of age

A child presents with a 5 mm wart on the lateral aspect of the fifth digit on the right hand. Which treatment is MOST appropriate? A. cryotherapy for 2-10 seconds B. oral cimetidine (Tagamet) C. salicylic acid D. surgical excision with scalpel

The correct answer is C - salicylic acid Verrucae vulgaris, referred to as common warts, are most typically caused by human papillomaviruses type 2 and 4. These warts occur frequently on fingers, the back of hands, in tissue folds around nails, and on the face, knees, and elbows. Approximately 5-10% of children develop warts, which are spread by direct contact and autoinoculation. Salicylic acid is an over-the-counter effective, low-cost, and painless intervention for treatment of warts. Cryotherapy is an alternative first-line agent for common warts but may be painful, and should not be used over joints or the lateral aspect of digits. Surgical excision can cause scarring and cause more pain than the wart itself. Oral cimetidine (Tagamet) is a histamine 2-receptor-blocking agent and may be used as adjunctive treatment for refractory warts by improving immunity to human papillomavirus

A preschooler has a history of severe asthma with six hospitalizations over the past two years. One parent states that the other parent continues to smoke around the child and requests that the other parent be reported to child protective services for neglect. Before filing such a report, it is MOST important to determine that the A. child has been shown on RAST testing to be allergic to secondhand smoke. B. child has had a prior reaction of wheezing when exposed to smoke. C. smoking parent has been adequately educated as to the danger of secondhand smoke. D. nonsmoking parent has been adequately educated as to the danger of secondhand smoke.

The correct answer is C - smoking parent has been adequately educated as to the danger of secondhand smoke. Exposure of a child with asthma to smoke, a well-known environmental hazard and trigger of asthma, can be a manifestation of possible child neglect. In this case, the most important issue for assessing suspected child neglect is the smoking parent's understanding of the hazards associated with smoking and the child's asthma. Prior to reporting the smoking parent to child protective authorities, the health care provider needs to determine what efforts have been made to educate the smoking parent about the dangers of smoking, especially around the child. In addition, it is important to determine what efforts have been made to help the smoking parent access smoking cessation resources and knowledge of alternate preventive strategies such as smoking outside the home. Even if the child had not been admitted to a pediatric critical care unit or was never intubated, there would still be a concern of neglect related to continued exposure to smoke. The nonsmoking parent understands the effects of smoke on asthma and they are not based on an actual allergy to the smoke. The first step in the management of this case is to intervene with the smoking parent directly and assess understanding of the situation and prevention strategies. If that fails, child protective services may need to be involved

An adolescent with primary hypertension has left ventricular hypertrophy identified on echocardiogram. The appropriate therapeutic goal for the blood pressure is a reduction A. of diastolic value by 10 mmHg. B. of systolic value by 10 mmHg. C. to less than the 90th percentile for gender, age, and height. D. to at least the 95th percentile for gender, age, and height.

The correct answer is C - to less than the 90th percentile for gender, age, and height. The initial approach to primary hypertension in children and adolescents is weight loss. Initial management is instituted with planned, regular follow-up based on severity of hypertension and body mass index. While weight loss has been shown to correlate with a decrease in blood pressure, it requires education and participation from both the child and the family. Physical activity is inversely related to blood pressure and is utilized in prevention as well as management of hypertension; however, those with stage II hypertension, should not participate in strenuous activity until a normal blood pressure can be achieved. Recommendations set forth in the 2017 American Academy of Pediatrics (AAP) Clinical Practice Guideline for Screening and Management of High Blood Pressure in Children and Adolescents are to decrease the blood pressure to less than the 95th percentile in those with uncomplicated primary hypertension. For those with concurrent disease or target organ damage, the recommended goal for blood pressure is less than the 90th percentile. In the scenario provided, left ventricular hypertrophy is evidence of target organ damage. Other examples of target organ damage include retinal vascular changes and impaired renal function. Referral to appropriate subspecialists is recommended for children with secondary hypertension and those with target organ damage, as well as those who do not achieve goal blood pressure after lifestyle modification and 6 months of dual pharmacotherapy. Referral should be considered for children with stage II hypertension or multiple obesity-related conditions

The MOST common presentation of mild fluorosis is A. pitting of primary teeth. B. dark staining of permanent teeth. C. gray discoloration of primary teeth. D. chalky white lines on permanent teeth

The correct answer is D - chalky white lines on permanent teeth. Fluoride prevents dental caries primarily through its topical effect on teeth, and secondarily by the incorporation of systemic fluoride into the enamel of developing teeth. Most children whose water supply is fluoridated require no additional dietary supplementation of fluoride. Breast fed infants generally require fluoride supplementation from 6 months of age until they are taking adequate amounts of fluoride in drinking water. The American Dental Association has developed an algorithm indicating recommended fluoride supplement dosing for infants and children with reference to the level of fluoride in the child's drinking water. Water may be analyzed by most local health departments for fluoride content. Fluoride is also ingested by children taking fluoride supplements, drinking some kinds of bottled waters and juices, and swallowing fluoride toothpaste. Excessive fluoride intake interferes with enamel formation, causing fluorosis. Mild fluorosis presents as chalky white lines on the cutting edges of front permanent teeth. More severe fluorosis causes dark staining and pitting of permanent teeth. Evidence of fluorosis is not usually seen in deciduous teeth. Yellowing of teeth is usually the result of antibiotic therapy such as tetracycline. Gray discoloration can result from oral iron supplements or could be associated with trauma to the tooth resulting in lack of blood flow.

When obtaining a comprehensive health history for a new adolescent patient, which is the BEST approach for the psychosocial assessment? A. refrain from asking sexuality related questions unless the caregiver has concerns B. ask the caregiver to complete an anxiety screening questionnaire C. begin the psychosocial screening with personal questions D. complete the history without the caregiver present

The correct answer is D - complete the history without the caregiver present The psychosocial history for an adolescent should be completed without the caregiver present to facilitate the discussion of sensitive topics with the health care provider. The health care provider explains at the start of the visit that the adolescent will be interviewed separately to determine how things are going at home, school and with friends. Even if the caregiver has concerns, rapport with the adolescent needs to be established before asking invasive and personal questions. To complete the psychosocial history, a key component of an adolescent's health history, health care providers often use the HEEADSSS or SSHADESS mnemonics (see table below) to approach the history using open-ended questions

When assessing social language and self-help domains for a healthy 18 month old, which of the following findings would be a concern? A. frequently brings objects to caregiver B. points to pictures while being read a book C. assists with dressing and undressing by raising arms D. has no interest when caregiver begins a new activity

The correct answer is D - has no interest when caregiver begins a new activity Social referencing, when an infant looks to the primary caregiver for signals on how to deal with new experiences, and joint attention, the ability to coordinate attention with others regarding objects and events, should be developed by 18 months of age. By 18 months children can be expected to bring and show objects to adults, help with getting dressed/undressed, and point to pictures in books. Therefore, it would be concerning for an 18 month old to be uninterested when the caregiver engages in a new activity. Impairments in joint attention and social referencing are associated with autism spectrum disorder

Which laboratory testing is recommended annually for an adolescent with Turner syndrome? A. CBC, serum glucose, celiac panel B. celiac panel, CBC, thyroid function C. CBC, celiac panel, metabolic panel D. lipid profile, serum glucose, thyroid function

The correct answer is D - lipid profile, serum glucose, thyroid function Among females with Turner syndrome, there is a high level of undiagnosed lipid and thyroid abnormalities. Glucose intolerance is common, and hypothyroidism occurs frequently in children with Turner syndrome. Therefore, clinical guidelines endorsed by the American Academy of Pediatrics (AAP) include annual blood pressure check, fasting lipids, and blood glucose, as well as liver and thyroid function tests. Children with Turner syndrome have an increased risk for celiac disease and thus screening is recommended beginning at 2-3 years of age and at a frequency of every 2 years throughout childhood. Renal function, metabolic profile, and cardiology evaluation should be obtained at the time of diagnosis. ENT and audiology assessments should also be obtained every 1-5 years.

Screening for depression among adolescents should occur: A. annually during patient interview B. at every visit using a formal self-reported screening tool C. annually using a parent-completed Pediatric Symptom Checklist D. more often when there are frequent somatic complaints

The correct answer is D - more often when there are frequent somatic complaints Although as many as one in five adolescents experiences depression, the condition often goes undetected. The poor rates of detection and treatment of adolescent depression can be partially attributed to the lack of access to mental health providers. For this reason, pediatric primary care providers play an active role in identification and support of adolescents with depression. The current recommendation is that a universal screening for depression should occur annually in all adolescents 12 years of age or older using a formal self-report screening tool. Although it is acceptable to use a parent completed Pediatric Symptom Checklist (PSC), the screen is best accomplished by using a formal self report screening tool. Patient interview is valuable, but given the high prevalence of adolescent depression, universal screening may result in increased recognition and treatment of depressive disorders in adolescents. Targeted depression screening, using a formal depression instrument or tool, should occur more frequently when depression risk factors are present to include a history of previous depressive episodes, family history, other psychiatric disorders, substance use, trauma, psychosocial adversity, frequent somatic complaints, or previous screening tools with high scores without a diagnosis of depression

A school-age child presents with 2 days of loose stools preceded by a 2-week history of hard stools and pain with defecation. A hard mass is palpable in the left lower quadrant on exam. Which of the following medications is MOST appropriate? A. azithromycin (Zithromax) B. bismuth subsalicylate C. loperamide D. polyethylene glycol

The correct answer is D - polyethylene glycol The history and physical exam of this child are consistent with a diagnosis of encopresis. The appropriate treatment for this condition is a bowel cleanout. Soiling will not resolve without disimpaction. Recommendations should include administering oral cathartics at bedtime 1-3 nights in a row and 1 fleet enema twice daily for 1-3 days in a row. Polyethylene glycol is an appropriate treatment. Bismuth subsalicylate and loperamide are both anti-diarrheal agents and would worsen the impaction. Additionally, bismuth subsalicylate is approved for children ages 12 years and older. It contains aspirinlike ingredients and should not be used in children with chicken pox or flu-like symptoms due to the risk for Reyes syndrome. Azithromycin (Zithromax) is an antibiotic that is sometimes used to treat traveler's diarrhea, but there is no evidence from the history or physical that this is the etiology for the child's loose stools

A preschooler with type 1 diabetes and no gastrointestinal symptoms has an elevated tissue transglutaminase IgA on routine screening. The NEXT step is to A. order HLA testing. B. begin gluten free diet. C. order intestinal biopsy. D. refer to a gastroenterologist.

The correct answer is D - refer to a gastroenterologist. Celiac disease is an autoimmune disease that occurs in genetically predisposed individuals. In celiac disease, ingestion of gluten from wheat, rye, or barley damages the small intestine. The more recent availability of serologic testing has shown that celiac disease occurs throughout the world, with the highest prevalence in the Saharawi population of Western Sahara Africa and in Spain. The suggested prevalence in the U.S., Europe and other countries is 1:100. Despite the availability of testing, the condition remains largely undiagnosed. Testing children for celiac disease is recommended by current evidence-based guidelines for those with typical gastrointestinal (GI) manifestations of diarrhea, weight loss, and abdominal distention, as well as for those with less typical GI manifestations that have no other identified cause. For children with identified risk factors of type 1 diabetes and first-degree relatives with the disease, the guidelines from the North American Society for Pediatric Gastroenterology, Hepatology and Nutrition (NASPGHAN) recommend testing beginning at age 3 years provided the child's diet contains gluten. Other risk factors identified in the NASPGHAN guidelines as indications for testing of asymptomatic children include dermatitis herpetiformis, IgA deficiency, Down syndrome, Turner syndrome, and Williams syndrome. Children with positive serologic markers for celiac disease should be referred to a gastroenterologist for confirmatory testing. According to the NASPGHAN guideline, a number of issues may make interpretation of test results challenging, including how the tests were evaluated for sensitivity and specificity, changes in technical aspects of performing the tests over time, and importantly in this scenario, limited data on serologic testing in children under 5 years of age. Intestinal biopsy may be needed in addition to serologic testing in some children. Definitive diagnosis is best made by a specialist in pediatric gastroenterology.

Education for caregivers about safe use of an infant car safety seat includes that the A. harness straps should be above the shoulders with the seat rear-facing. B. seat should shift a little more than 1 inch from side to side. C. infant can be swaddled prior to placing in the seat. D. retainer clip should be at armpit level.

The correct answer is D - retainer clip should be at armpit level. Motor vehicle collisions are the leading cause of death for children in all age groups. A child restraint system (CRS) should be installed tightly to decrease movement of the system itself. When the seat is grasped at the bottom near the belt or lower anchors and moved, it should move no more than an inch. More than that amount of movement increases the probability of injury during an accident. Infants and children should not be swaddled, wrapped in blankets, or wear heavy coats when placed in a CRS as these will compress during a collision, allowing for movement and increasing the risk of injury. Instead, place the infant in the car seat as recommended, and then place a blanket over the child. This allows the straps to be tight holding the child or infant in place. An infant should be in a rear-facing car seat until 2 years of age or until they have reached the weight or length limits as indicated by the manufacturer. The retainer clip should be at the armpit level to help hold the upper body in place in the event of a collision. Harness straps should be inserted into the slots that are at or below the level the infants' shoulders. Positioning infants and children correctly in child restraint systems can prevent fatalities from motor vehicle trauma. Car safety seats must be installed tight enough in order to provide protection for an infant during a vehicle crash.

Counseling regarding meningococcal disease for a prospective college freshman who plans to live in a dormitory should include that the A. meningococcal vaccine confers lifelong immunity. B. meningococcal vaccine is mandatory for all college students. C. risk for meningococcal disease is just as high for commuting students. D. risk for meningococcal disease is increased for freshmen dormitory residents.

The correct answer is D - risk for meningococcal disease is increased for freshmen dormitory residents. The Advisory Committee on Immunization Practices (ACIP) recommends that prospective college freshmen, who will be living in dormitories, and their parents, be counseled regarding the risks and benefits of meningococcal vaccination. The decision to vaccinate lies with the parent and adolescent after counseling and is not mandatory, although some colleges are now requiring vaccination for admission to dormitory living. The vaccine protects against approximately 50% of the currently circulating serotypes for meningococcal meningitis, and the duration of protection is 3 - 5 years. Meningococcal disease risk is increased for freshmen living in dorms compared to the same age population not in dormitory living. For students other than freshmen the disease risk is similar to the general population

An adolescent complains of shortness of breath, coughing, throat tightness, and audible laryngeal wheezing with stridor during soccer games. Spirometric lung function testing reveals "truncated" and inconsistent inspiratory and expiratory flow-volume loops. Referral should be made to a/an: A. occupational therapist B. physical therapist C. psychologist D. speech therapist

The correct answer is D - speech therapist In older children and adolescents, vocal cord dysfunction (VCD) can manifest as intermittent daytime wheezing. In this condition, the vocal cords close involuntarily and inappropriately during inspiration and at times, exhalation, producing shortness of breath, coughing, throat tightness, and audible wheezing and/or stridor. In most cases of VCD, spirometric lung function testing reveals "truncated" and inconsistent inspiratory and expiratory flow-volume loops; lung function does not improve with bronchodilators. Flexible laryngoscopy demonstrates anatomically normal but paradoxical vocal cord movements. This condition can be managed with specialized speech therapy training in the relaxation and control of vocal cord movement

A pediatric nurse practitioner is considering leaving primary care for a position in a mental health clinic which includes screening and managing medications for patients of all ages. Before making such a change in employment it is MOST important to consult the A. staff at the certification board. B. staff at the malpractice insurance carrier. C. professional organization's scope and standards of practice. D. state board of nursing's rules and regulations.

The correct answer is D - state board of nursing's rules and regulations. Nurse practitioners should be aware of their state board of nursing's rules and regulations because these are what determine legal practice parameters. It would be prudent to check with the insurance carrier, as well. Certification boards assure the public an individual has a certain level of knowledge in a specialty area but do not set age parameters for practice. Membership organizations provide an arena for continuing education and suggestions for how to practice in that specialty. Consistent with the APRN Consensus Model of 2008, it is also essential to consider whether or not the nurse practitioner has the education, certification and licensure to prepare her or him to evaluate patients of a particular age or population. The pediatric nurse practitioner in this example has presumably not met the academic or certification requirements to evaluate and treat adults.

Which of the following is MOST appropriately included in guidance provided to caregivers about reducing their adolescent's risk of dating violence? A. encourage abstinence from sexual intercourse B. monitor for school problems and depression C. review text messages and emails D. talk about healthy relationships

The correct answer is D - talk about healthy relationships Dating violence may start as early as middle school. It can involve physical or emotional abuse, and the violent partner may be male or female. A violent partner may be described as moody, volatile, bossy, controlling or jealous. The victim of this type of behavior is at risk for depression, substance abuse, and school problems. Preventing dating violence is key. Parents should be encouraged to talk to their children about healthy relationships and should act as role models for their children. People in healthy relationships respect and encourage each other, share interests, have friends outside of the relationship, and calmly settle differences

A term infant born by C-section, with birthweight of 5 lb 5 oz (2.4 kg), develops a respiratory rate of 68 bpm with clear breath sounds at 2 hours of life. The MOST likely diagnosis is: A. congenitally acquired pneumonia B. pneumothorax incident to delivery C. respiratory distress syndrome D. transient tachypnea of the newborn

The correct answer is D - transient tachypnea of the newborn Transient tachypnea of the newborn (TTN) results from fluid remaining in the lungs of a newborn after the birth process. It is more common in term infants born by cesarean delivery. Symptoms usually begin shortly after birth and include rapid respiratory rate without hypoxia or with mild hypoxia resolving with the use of oxygen. Infants can also have symptoms of grunting and mild retractions despite clear lung sounds upon auscultation. Symptoms of TTN usually resolve by 3 days of life. Respiratory distress syndrome is associated with prematurity or other birth related problems. Symptoms include significant hypoxia, diminished air entry on auscultation and progressive respiratory distress over the first few hours of life. A pneumothorax resulting from the delivery process would cause tachypnea, but may also present with decreased breath sounds on the affected side and shifted heart sounds. The infant may also require supplemental oxygen. An infant with congenitally acquired pneumonia would present with symptoms either at birth or within a few hours of birth, but will most likely have abnormal breath sounds and an oxygen requirement.

Adolescents who receive education about condoms should also have information about the A. usage of petroleum jelly for lubrication. B. brands least likely to break during intercourse. C. technique for safe removal focusing on rerolling condom. D. use of long-acting reversible contraception as dual-protection

The correct answer is D - use of long-acting reversible contraception as dual-protection. Condoms are the most frequently used method of birth control among adolescents. They are low in cost, available without a prescription, and require little advanced planning. They also prevent the transmission of HIV, human papillomavirus, and other sexually transmitted infections (STIs). However, they do carry a failure rate of approximately 18%. Therefore, it is highly recommended that adolescents use dual protection. Counseling should include the use of condoms to prevent STIs in addition to a highly effective birth control method such as long-acting reversible contraception. After sex to keep the condom in place it should be held at the base while pulling out. Once withdrawn, the condom can be removed from the penis using care to contain contents before discarding


संबंधित स्टडी सेट्स

American History Vocabulary Unit 5.2

View Set

Exam 1 History of World Architecture III

View Set

Final Exam: Renal Dysfunction NCLEX Questions

View Set

Oceanography Chapter 4: Marine Sediments

View Set

Classical & Operant Conditioning

View Set

INFORMATION SECURITY MIDTERM EXAM QUESTIONS

View Set

Final The Interior of the Earth and Introductory to Plate Tectonics

View Set